Verbal review, #69 V.Good Strengthen Qs

This topic has expert replies
Master | Next Rank: 500 Posts
Posts: 122
Joined: Mon Dec 15, 2008 12:03 am
Thanked: 2 times

Verbal review, #69 V.Good Strengthen Qs

by vkb16 » Wed Mar 25, 2009 9:20 pm
Manufacturers sometimes discount the price of a product to retailers for a promotion period when the product is advertised to consumers. Such promotions often result in a dramatic increase in amount of product sold by the manufacturers to retailers. Nevertheless, the manufacturers could often make more profit by not holding the promotions.

Which of the following, if true, most strongly supports the claim above about the manufacturers' profit?

(A) The amount of discount generally offered by manufacturers to retailers is carefully calculated to represent the minimum needed to draw consumers' attention to the product.
(B) For many consumer products the period of advertising discounted prices to consumers is about a week, not sufficiently long for consumers to become used to the sale price.
(C) For products that are not newly introduced, the purpose of such promotions is to keep the products in the minds of consumers and to attract consumers who are currently using competing products.
(D) During such a promotion retailers tend to accumulate in their warehouses inventory bought at discount; they then sell much of it later at their regular price.
(E) If a manufacturer falls to offer such promotions but its competitor offers them, that competitor will tend to attract consumers away from the manufacturer's product.

OA is D
I say B

why should the OA be right? I have two reasons why OA cant be right
1. The answer is about retailers, so it doesnt affect the conclusion about MANUFACTURERS, and
2. Even IF the retailers make more profir selling it later, how is manufacturers' profits affected? Isnt mfg's profit only connected with its selling price to retailers? for the retailers, mfg's selling price is retailer's cost price.

User avatar
Legendary Member
Posts: 682
Joined: Fri Jan 16, 2009 2:40 am
Thanked: 32 times
Followed by:1 members

Re: Verbal review, #69 V.Good Strengthen Qs

by Vemuri » Thu Mar 26, 2009 4:07 am
I read in Princeton review guide that the best way to address these type of questions is to eliminate options which are definitely wrong. The option that is left is the best answer to the question. Observe how the question is framed "Which of the following, if true, most strongly supports the claim.."

Option B actually weakens the argument.

Thanks for posting such an interesting CR

Master | Next Rank: 500 Posts
Posts: 115
Joined: Thu Nov 27, 2008 3:17 am
Thanked: 1 times

by kartik1979 » Thu Mar 26, 2009 5:01 am
doesnt B state that manufactures dont have to do any promotions and thus not incur th cost associated with it . Hence they will benefit by selling at usual price

D states that sales happen to retailers at lower price and retailers sell them at normal price later but retailers only buy huge quanties as there is discount offered to them, if discount is not offered to them they will buy normal quantity at regular price

Still confused over both these options can anyone pls state why D is better than b

Junior | Next Rank: 30 Posts
Posts: 26
Joined: Fri Feb 13, 2009 6:23 am
Location: frankfurt
GMAT Score:620

by usualsuspect » Thu Mar 26, 2009 6:02 am
The passage says that promotions leads to increase in sales. But it also says that this does not lead to increase in profit for the manufacturer.
The passage also says that discounts are given to the retailers.

D explains that if the retailers hoard the supplies and sell at regular price, the manufacturer is at loss. This wud greatly strengthen the conclusion that "promotions lead to no increase in profit for manufacturers".
Since the demand for the products is present at regular price levels, it makes no sense to give discount to the retailers.
Aëtou gēras, korydou neotēs.
"An eagle's old age (is worth) a sparrow's youth".

Master | Next Rank: 500 Posts
Posts: 122
Joined: Mon Dec 15, 2008 12:03 am
Thanked: 2 times

by vkb16 » Thu Mar 26, 2009 6:28 am
The passage says that promotions leads to increase in sales. But it also says that this does not lead to increase in profit for the manufacturer.
The passage also says that discounts are given to the retailers.

D explains that if the retailers hoard the supplies and sell at regular price, the manufacturer is at loss. This wud greatly strengthen the conclusion that "promotions lead to no increase in profit for manufacturers".
Since the demand for the products is present at regular price levels, it makes no sense to give discount to the retailers.
D explains that if the retailers hoard the supplies and sell at regular price, the manufacturer is at loss.
usualsuspect, how does retailers selling at regular price (to consumers) affect wholesalers (independent of conumers)?

Junior | Next Rank: 30 Posts
Posts: 26
Joined: Fri Feb 13, 2009 6:23 am
Location: frankfurt
GMAT Score:620

by usualsuspect » Thu Mar 26, 2009 6:58 am
Lets assume the actual price the manufacturer sells the product to the retailer = 100$. Now he gives the retailer 20% discount, so his selling price is now 80$.
Now, the manufacturer is under the assumption that this discount is directly passed on to the customer. and this discount would increase his sales. Now, the sales actually does increase.
But the argument concludes that " Nevertheless, the manufacturers could often make more profit by not holding the promotions."

in D, the retailer is selling at regular price (100$) and not at 80$. This directly strengthens the argument that a promotion is not required.( Remember we have to choose a choice that strongly supports the conclusion). The unstated assumption in this argument is, there is some other factor which boosts the sales. Maybe brand image etc...
Aëtou gēras, korydou neotēs.
"An eagle's old age (is worth) a sparrow's youth".

Master | Next Rank: 500 Posts
Posts: 122
Joined: Mon Dec 15, 2008 12:03 am
Thanked: 2 times

by vkb16 » Tue Mar 31, 2009 3:01 am
thanks.. i guess the hint is at the opportunity cost concept! thanks a lot!

Master | Next Rank: 500 Posts
Posts: 100
Joined: Tue Aug 05, 2014 11:00 am

by samanthaJ79 » Sun May 15, 2016 5:12 am
I guess B is right

Master | Next Rank: 500 Posts
Posts: 108
Joined: Tue Aug 05, 2014 3:00 am

by graem83d » Sun May 15, 2016 7:23 am
I feel the answer will be B